x^2 + mx + n = 0, m and n are integers. The only possible value for x is -3. What is the value of m?

plz the solution is necessary

Thanks​

Answers

Answer 1

Answer:

The value of m is 3

Step-by-step explanation:

we have

[tex]x^{2}+mx+n=0[/tex]

step 1

Find the value of n

For x=0

[tex]0^{2}+m(0)+n=0[/tex]

[tex]n=0[/tex]

step 2

Find the value of m

For x=-3

substitute

[tex]x^{2}+mx=0[/tex]

[tex](-3)^{2}+m(-3)=0[/tex]

[tex]9-3m=0[/tex]

[tex]3m=9[/tex]

[tex]m=3[/tex]


Related Questions

Find the inverse function of g(x) = 2x + 4. g -1(x) = 4x + 2 g -1(x) = 2x + g -1(x) = x - 2 g -1(x) = 2x - 4

Answers

For this case we must find the inverse of the following function:

[tex]g (x) = 2x + 4[/tex]

Replace g(x) with y:

[tex]y = 2x + 4[/tex]

We exchange the variables:

[tex]x = 2y + 4[/tex]

We solve for "y":

We subtract 4 on both sides of the equation:

[tex]x-4 = 2y[/tex]

We divide between 2 on both sides of the equation:

[tex]y = \frac {x} {2} -2[/tex]

We change y by [tex]g ^ {-1} (x)[/tex]:

[tex]g ^ {- 1} (x) = \frac {x} {2} -2[/tex]

Answer:

[tex]g ^ {- 1} (x) = \frac {x} {2} -2[/tex]

Answer:

[tex]f^{-1}=\frac{x}{2} -2[/tex]

Step-by-step explanation:

the inverse function of g(x) = 2x + 4

To find the inverse of a function we replace g(x) with y

[tex]y=2x+4[/tex]

Replace x with y and y with x

[tex]x=2y+4[/tex]

Solve the equation for y

Subtract 4 from both sides

[tex]x-4= 2y[/tex]

Divide both sides by 2

[tex]\frac{x}{2} -2=y[/tex]

[tex]y=\frac{x}{2} -2[/tex]

Replace y with f inverse

[tex]f^{-1}=\frac{x}{2} -2[/tex]

Find the length of arc YXZ.

A. s=3
B. s= 60
C. s= 75
D. s= 236

Answers

Answer:

C. s= 75

Step-by-step explanation:

One radian is the angle subtended by an arc length equal to the radius of the circle.

∅=s/r where s is the arc length ∅ the angle and r the radius of the circle.

Thus s=∅×r

In the circle provided arc s subtends and angle of 5 radians.

s= 5 radians× 15

=75

B. S=60 , dont know for sure

Write the equation in vertex form and then find the vertex, focus, and directrix of a parabola with equation x=y^2+18y-2

Answers

Answer:

Part 1) The vertex is the point (-83,-9)

Part 2) The focus is the point (-82.75,-9)

Part 3) The directrix is [tex]x=-83.25[/tex]

Step-by-step explanation:

step 1

Find the vertex

we know that

The equation of a horizontal parabola in the standard form is equal to

[tex](y - k)^{2}=4p(x - h)[/tex]

where

p≠ 0.

(h,k) is the vertex

(h + p, k) is the focus

x=h-p is the directrix

In this problem we have

[tex]x=y^{2} +18y-2[/tex]

Convert to standard form

[tex]x+2=y^{2} +18y[/tex]

[tex]x+2+81=y^{2} +18y+81[/tex]

[tex]x+83=(y+9)^{2}[/tex]

so

This is a horizontal parabola open to the right

(h,k) is the point (-83,-9)

so

The vertex is the point (-83,-9)

step 2

we have

[tex]x+83=(y+9)^{2}[/tex]

Find the value of p

[tex]4p=1[/tex]

[tex]p=1/4[/tex]

Find the focus

(h + p, k) is the focus

substitute

(-83+1/4,-9)

The focus is the point (-82.75,-9)

step 3

Find the directrix

The directrix of a horizontal parabola is

[tex]x=h-p[/tex]

substitute

[tex]x=-83-1/4[/tex]

[tex]x=-83.25[/tex]

PLZ HURRY IT'S URGENT!
Mr. Caldwell has 28 students in his class and 12 of them are boys. What is the ratio of girls to boys? A. 4 : 7 B. 3 : 4 C. 4 : 3 D. 7 : 4

Answers

Answer:

C

Step-by-step explanation:

Girls - 28-12 = 16

16:12 = 4:3 - so answer choice C

We know that out of the 28 students 12 of them are boys. To find how many girls there are subtract 12 from 28

28 - 12 = 16

That means that there are 16 girls in the class

Now you need a ratio of girls : boys so...

16 : 12

This can be further simplified to the following...

4 : 3 (C)

This means that for every four girls there are 3 boys

Hope this helped!

~Just a girl in love with Shawn Mendes

What is 6 consecutive integers starting with -3

Answers

Answer:

-3, -2, -1, 0, 1, 2

Step-by-step explanation:

Note that "integer" means whole numbers, and "consecutive" means continuously following. Also, note that you are starting with -3:

-3 , -2 , -1 , 0 , 1 , 2, is your answer, because you just need to add 1 each time until you have 6 numbers.

~

Which describes the scatter plot of a car’s value compared to the age of the car ?

Answers

So the x-axis represents the age of the car and the y represents the value of the car

As you can see as the x values increase the y value decreases

This means that the answer is C. As the age of the car increases the value of the car decreases

Hope this helped!

~Just a girl in love with Shawn Mendes

Answer:

3

Step-by-step explanation:

Graph 2x -2= -4 and solve

Answers

Answer:

x=-1

Step-by-step explanation:

2x -2= -4

2x = -2

x = -1

Answer:

-1

Step-by-step explanation:

Assume x > 0 and y 0.

algebra II engenuity

Answers

Answer:

Third Option

Step-by-step explanation:

Given expression is:

[tex]\sqrt{\frac{55x^7y^6}{11x^11y^8}}[/tex]

Simplifying

[tex]=\sqrt{\frac{11*5*x^7*y^6}{11*x^11*y^8}}\\=\sqrt{\frac{5*x^7*y^6}{x^11*y^8}}\\Combining\ exponents\ with\ same\ base\\=\sqrt{\frac{5}{11*x^{(11-7)}*y^{(8-6)}}}\\=\sqrt{\frac{5}{x^{4}*y^{2}}}\\Applying\ radical\\=\frac{5^{\frac{1}{2}}}{x^{(4*\frac{1}{2})}*y^{(2*\frac{1}{2})}}}\\=\frac{\sqrt{5}}{x^2y}[/tex]

As 5 cannot be taken out of the square root, it will remain inside the square root.

Hence, Third option is the correct answer ..

find the exact value of cos(sin^-1(-5/13))

Answers

bearing in mind that the hypotenuse is never negative, since it's just a distance unit, so if an angle has a sine ratio of -(5/13) the negative must be the numerator, namely -5/13.

[tex]\bf cos\left[ sin^{-1}\left( -\cfrac{5}{13} \right) \right] \\\\[-0.35em] ~\dotfill\\\\ \stackrel{\textit{then we can say that}~\hfill }{sin^{-1}\left( -\cfrac{5}{13} \right)\implies \theta }\qquad \qquad \stackrel{\textit{therefore then}~\hfill }{sin(\theta )=\cfrac{\stackrel{opposite}{-5}}{\stackrel{hypotenuse}{13}}}\impliedby \textit{let's find the \underline{adjacent}}[/tex]

[tex]\bf \textit{using the pythagorean theorem} \\\\ c^2=a^2+b^2\implies \pm\sqrt{c^2-b^2}=a \qquad \begin{cases} c=hypotenuse\\ a=adjacent\\ b=opposite\\ \end{cases} \\\\\\ \pm\sqrt{13^2-(-5)^2}=a\implies \pm\sqrt{144}=a\implies \pm 12=a \\\\[-0.35em] ~\dotfill\\\\ cos\left[ sin^{-1}\left( -\cfrac{5}{13} \right) \right]\implies cos(\theta )=\cfrac{\stackrel{adjacent}{\pm 12}}{13}[/tex]

le's bear in mind that the sine is negative on both the III and IV Quadrants, so both angles are feasible for this sine and therefore, for the III Quadrant we'd have a negative cosine, and for the IV Quadrant we'd have a positive cosine.

Solve the following inequality-3.55g<-28.4

Answers

Answer:

B  (the second graph)

Step-by-step explanation:

3.55g<-28.4

Divide each side by 3.55

3.55g/3.55<-28.4/3.55

g < -8

This would be an open circle at 8  (because g is less than not less than or equal to)  and it goes to the left ( since g is less than)

Use the given parent function f(x) = |x| to graph g(x) = |x| -4.
Use the ray tool and select two points to graph each ray.
Can somebody help me please I hate graphs

Answers

Answer:

Look to the attached graph

Step-by-step explanation:

* Lets explain the difference between the graphs of f(x) and g(x)

∵ f(x) = IxI

∵ g(x) = IxI - 4

- If we add are subtract f(x) by k, where k is a constant that means

 we translate f(x) vertically

- If g(x) = f(x) + k

∴ f(x) translated vertically k units up

- If g(x) = f(x) - k

∴ f(x) translated vertically k units down

∵ g(x) = IxI - 4

∵ f(x) = IxI

∴ g(x) = f(x) - 4

∴ f(x) translated vertically 4 units down

∴ The graph of f(x) will translate down 4 units

∵ The origin point (0 , 0) lies on f(x)

∴ The origin point (0 , 0) will translate down by 4 units

∴ Its image will be point (0 , -4)

∴ Point (0 , -4) lies on the graph of g(x)

- So you can translate each point on the graph of f(x) 4 units down to

 graph g(x)

# Two point on the left part

∵ Point (-2 , 2) lies on f(x)

∴ Its image after translation 4 units down will be (-2 , -2)

∴ Point (-2 , 2) lies on g(x)

∵ Point (-7 , 7) lies on f(x)

∴ Its image after translation 4 units down will be (-7 , 3)

∴ Point (-7 , 3) lies on g(x)

# Two point on the right part

∵ Point (3 , 3) lies on f(x)

∴ Its image after translation 4 units down will be (3 , -1)

∴ Point (3 , -1) lies on g(x)

∵ Point (8 , 8) lies on f(x)

∴ Its image after translation 4 units down will be (8 , 4)

∴ Point (8 , 4) lies on g(x)

* Now you can draw the graph with these 5 points

sin θ > 0 and tan θ < 0 then:

Answers

Answer:

last choice

Step-by-step explanation:

sin(theta) is positive

tan(theta)=sin(theta)/cos(theta) is negative

If sin(theta) is positive and tan(theta) is negative, then cos(theta) is negative because +/-=-

So we are looking for the quadrant where sine is positive and cosine is negative or when x is negative and y is positive.

This is in quadrant 2 so our angle theta is between 90 and 180 degrees.

which point on the number line represent the product of (-4)(-2)(-1)

Answers

-4 times -2 equals 8.

8 times -1 equals -8.

Therefore, the point on your number line that is on -8 is your answer!

4x2=8 si your answer -8 is correct

Remove the largest possible common factor check your answer by multiplication
[tex]10x {}^{2} - 15x[/tex]


A.
[tex]10x {}^{2}(5x - 3)[/tex]
B.
[tex]5x(2x - 3)[/tex]
C.
[tex] - 5x(2x + 3)[/tex]
D.
[tex]5x(2x + 3)[/tex]

Answers

The greater common divisor between 10 and 15 is 5, so we can factor it out:

[tex]10x^2-15x = 5(2x^2-3x)[/tex]

The greater common exponent between 1 and 2 is 1, so we can factor x out:

[tex]10x^2-15x = 5x(2x-3)[/tex]

One cell phone plan charges $20 per month plus $0.15 per minute used. A second cell phone plan charges $35 per month plus $0.10 per minute used. Write and solve an equation to find the number of minutes you must talk to have the same cost for both calling plans.

Answers

Both of these equations are linear, so you have to write equations in the y=my+b format. And then you have to set the equations to equal each other to find x


Answer: 300 minutes

The equations are as follows where x represents the number of minutes the cell phone is used.

For plan one: Total cost = $20 + $0.15x

For plan two: Total cost = $35 + $0.10x

For both the costs to be the same, we need to use the cell phone for

300 minutes.

What are equations?

Equations are relations showing the value of one quantity related to another quantity when it can change. The changing value is the variable.

How do we solve the given question?

We are informed that one cell phone plan charges $20 per month plus $0.15 per minute used. A second cell phone plan charges $35 per month plus $0.10 per minute used.

We are asked to write and solve an equation to find the number of minutes you must talk to have the same cost for both calling plans.

Let the number of minutes the cell phone is used be x minutes.

Now we solve for equations for both plans in the following way:-

Plan one:

Charges $20 per month plus $0.15 per minute used.

When the use is for x minutes, the additional charge = $0.15*x = $0.15x

∴ Total cost = Fixed cost + Additional cost

or, Total cost = $20 + $0.15x.

Plan two:

Charges $35 per month plus $0.10 per minute used.

When the use is for x minutes, the additional charge = $0.10*x = $0.10x

∴ Total cost = Fixed cost + Additional cost

or, Total cost = $35 + $0.10x.

We are asked to find the number of minutes used so that the costs in both the plans are equal. To find this we equate the equation of total costs in both the cases to get:

$20 + $0.15x = $35 + $0.10x.

Subtracting ($20 + $0.10x) from both sides of the equation, we get

$20 + $0.15x - ($20 + $0.10x) = $35 + $0.10x - ($20 + $0.10x).

or, $20 + $0.15x - $20 - $0.10x = $35 + $0.10x - $20 - $0.10x.

or, $0.05x = $15

Dividing both sides of the equation by $0.05, we get

$0.05x/$0.05 = $10/$0.05

or, x = 300.

∴ We must talk for 300 minutes for both the plans to cost the same to us.

Learn more about equations at

https://brainly.com/question/2972832

#SPJ2

Which of the following is a polynomial?

Answers

Answer:

Option A and C are correct

Step-by-step explanation:

A polynomial is defined as an algebraic expression that consists of variables and constants involving operations of addition, subtraction, multiplication and non-negative exponents.

A. 4x^2+9x+12 is a polynomial

B. 3-√2x is not a polynomial

C. 15x^4 is a polynomial

D. 6x^2+2/x is not a polynomial

So, Option A and C are correct.

Answer: Option A and Option C

Step-by-step explanation:

Polynomials always have the following form:

[tex]ax ^ n + bx ^{n-1} + cx^{n-2} +...+ d[/tex]

Where a, b, c, and d are constant and are real numbers

The exponents of the variable x are always greater than or equal to zero.

Then identify among the options the expressions that have these characteristics

The answer are:

Option A.

[tex]4x^2 +9x+12[/tex]

Option C.

[tex]15x^4[/tex]

At a carnival, a toy boat race costs $4 to enter. Second place earns a prize worth $8, and third place earns a prize worth $5. If the probability of winning any of the prizes is [tex]\frac{1}{12}[/tex], what should the first place prize be worth for the game to be fair?



(a) $35


(b) $23


(c) $31


(d) $41

Answers

31 bro because if you think about it 8 times 4 is 26 + 5 more equals 31 c

The requried, to make the game fair, the first place prize should be worth 4n - 13 dollars. None of the options are correct.

What is probability?

Probability can be defined as the ratio of favorable outcomes to the total number of events.

Here,
The second place prize is worth $8 and the third place prize is worth $5, so the total value of these two prizes is $8 + $5 = $13.

If the game is fair, then the total amount of prize money awarded should be equal to the total amount of entry fees collected. If n players enter the game, then the total entry fees collected will be 4n.

Let x be the value of the first-place prize, which we want to find. The probability of winning any of the three prizes is 1/n, so the total amount of prize money awarded is,

1/n × x + 1/n × $8 + 1/n × $5 = ($4)n
x + 8 + 5 = 4n
x = 4n - 13

Therefore, to make the game fair, the first place prize should be worth 4n - 13 dollars.

Learn more about probability here:

https://brainly.com/question/24806777

#SPJ2

What’s this answer helppp‍♀️‍♀️

Answers

Step-by-step explanation:

formula of square= width x length

width = length

each sides = √ 64

= 8 cm

area of the rectangle = 1 & 1/2 x 64

= 96 cm²

formula for area of rectangle = " square

8 x length = 96 cm²

length= 96/8

= 12 cm

perimeter= 4 (8 + 12 + 12)

= 128 cm

*multiple with 4 cuz issa cross (??) i hope you get what i mean(??)

correct me if i'm wrong i'm learning too

Which equation shows the point slope form of the line that passes through (3, 2) and has a slope of
2
y + 2 =
(x + 3)
Oy-2=3(x-3)
y+3= }(x+2)
©y-3=2x-2)
Save and Exit
Markthandie

Answers

Answer:

y - 2 = 2(x - 3).

Step-by-step explanation:

The point -slope form of a line is:

y - y1 = m(x - x1) where m = the slope and (x1, y1) is a point on the line.

Here m = 2, x1 = 3 and y1 = 2. So we have the equation:

y - 2 = 2(x - 3).

Answer:

[tex]y=2x-4[/tex]

Step-by-step explanation:

Given: The line passes through (3, 2) and has a slope of  2.

To find: Point slope form of the equation.

Solution: We know that the point slope form of the line passing through [tex]\left ( x_{1},y_{1}\right )[/tex] and slope m is [tex]y-y_{1}=m\left ( x-x_{1} \right )[/tex]

Here, [tex]x_{1}=3,\ y_{1}=2, \text{and} \:\:m=2[/tex]  

So we have,

[tex]y-2=2(x-3)[/tex]

[tex]y-2=2x-6[/tex]

[tex]y=2x-4[/tex]

Hence, the point slope form of the line is [tex]y=2x-4[/tex].

Landry is building a rectangular flower garden. He wants the garden to be 3 feet longer than it is wide. He wants to buy a birdbath to put in the flower bed and after adding it he wants to have 35 square feet of space for planting. The cost of the birdbath will be $18 per square foot of the area of its base. He also needs to buy soil to cover the entire flower bed, including the space where the birdbath will be. The soil will cost $5 per square foot. He plans to spend a total of $150 on the soil and the birdbath. If x represents the width of the flower bed and y represents the area of the base of the birdbath, then which of the following systems of equations can be used to determine the length and width of the flower bed and the area of the base of the birdbath?

Answers

Answer:

Step-by-step explanation:

To create a system of equations, write an equation to model each condition that must be satisfied in the given situation.

It is given that x represents the width of the flower bed and that y represents the area of the base of the birdbath.

Write an equation to represent the condition regarding the planting area in the flower bed. It is given that there is 35 square feet of planting area. The flower bed is 3 feet longer than the width.

x(x+3) - y = 35

Write an equation to represent the condition regarding the cost of the soil and the birdbath. The question gives that the soil will cost $5 per square foot and the birdbath will cost $18 per square foot of the base.

5x(x+3)+18y=150

Combining both of the equations gives the following system of equations.

x(x+3)-y=35

5x(x+3)+18y=150

Solve for x.

x2 + 9x = 0

Answers

Answer:

0,-9

Step-by-step explanation:

To solve [tex]x^2[/tex] + 9x = 0, we factor out the common x to get x(x + 9) = 0 and set each factor equal to zero, which gives us the solutions x = 0 and x = -9.

To solve the equation x2 + 9x = 0, we can apply factoring techniques. The first step is to factor out the common factor x from both terms in the equation. This yields:

x(x + 9) = 0

Then you have to realize that a product of two multiplicands is equal to zero if either multiplicand is equal to zero. Setting either multiplicand equal to zero and solving for x yields the solutions. Therefore, we have two multiplicands:

x = 0

x + 9 = 0, which simplifies to x = -9

The solutions to the equation are x = 0 and x = -9. As a check, we can substitute these values back into the original equation and confirm that both satisfy the equation.

complete the synthetic division problem below x^3-2x^2-14x+3/x+3

Answers

Answer:

The steps are shown below. From here, we can write:

[tex]\frac{x^3-2x^2-14x+3}{x+3}=x^2-5x+1+\frac{0}{x+3} \\ \\ \therefore \boxed{\frac{x^3-2x^2-14x+3}{x+3}=x^2-5x+1}[/tex]

Answer:

x^2-5x+1

Step-by-step explanation:

this is correct

.The binomial (a + 5) is a factor of a2 + 7a + 10. What is the other factor?

a2 + 7a + 10 = (a + 5)(a +
)

Answers

Answer:

(a+2) Is your answer

Step-by-step explanation:

You could try synthetic division, but it's pretty complicated (not to mention for much harder problems), so you should try factoring this. What two numbers multiply up to 10 and add up to 7? Using trial and error, we would find these two numbers to be 2 and 5. Therefore, it factors into (a+5)(a+2).

Answer:

2

Step-by-step explanation:

Which is a true statement about the number 1

Answers

Your answer is the first option, "One is a factor of every whole number since every number is divisible by itself".

This is true because whenever you multiply 1 by anything you always get the thing you multiplied by, which makes it a factor of every number.

You can also find this by eliminating the other options, for example a prime number is any number whose factors are 1 and itself, 2 factors, so since 1 only has 1 factor it is not prime and option 3 is eliminated.

This also eliminates option 4 because 1 cannot be a composite number if it only has 1 factor.

I hope this helps!

The true statement among the given options is that one is a factor of every whole number since every number is divisible by itself.

What is a prime number?

A prime number is that number that is only fully divisible by 1 and that number itself.

Since we know that every number is divisible by 1. Thus 1 is also a factor of every number.

Thus one is a factor of every number.

For more about prime number

https://brainly.com/question/17748653

#SPJ2

What is the area of the parallelogram?

Answers

The area of a parallelogram is the base times the height.

Area = 14 x 8 = 112 square inches.

add 1/7 + -1/7 your answer should be reduced to its smallest form and should not be an improper fraction​

Answers

Answer: 0 is the answer, the 1/7 and -1/7 cancel each other out. Leaving zero as the answer!

What is the standard form equation of the line shown below?

Answers

Answer:

maybe

[tex] y = \frac{1}{2} x + 0.5[/tex]

Answer:

x - 2y = - 1

Step-by-step explanation:

The equation of a line in standard form is

Ax + By = C ( A is a positive integer and B, C are integers )

Obtain the equation in slope- intercept form

y = mx + c ( m is the slope and c the y- intercept )

Calculate m using the slope formula

m = (y₂ - y₁ ) / (x₂ - x₁ )

with (x₁, y₁ ) = (- 3, - 1) and (x₂, y₂ ) = (3, 2) ← 2 points on the line

m = [tex]\frac{2+1}{3+3}[/tex] = [tex]\frac{3}{6}[/tex] = [tex]\frac{1}{2}[/tex]

y = [tex]\frac{1}{2}[/tex] x + c ← is the partial equation

To find c substitute either of the 2 points into the partial equation

Using (3, 2), then

2 = [tex]\frac{3}{2}[/tex] + c ⇒ c = [tex]\frac{1}{2}[/tex]

y = [tex]\frac{1}{2}[/tex] x + [tex]\frac{1}{2}[/tex] ← in slope- intercept form

Multiply all terms by 2

2y = x + 1 ( subtract 2y from both sides )

0 = x - 2y + 1 ( subtract 1 from both sides )

- 1 = x - 2y , that is

x - 2y = - 1 ← in standard form

Please help me I’m terrible at math I’ll brainliest as soon as possible and if answer is right

Answers

Answer:

1.25

Step-by-step explanation:

Calculate the scale factor as the ratio of the corresponding sides of the image to the original.

here the image is 6.25 and the original is 5

scale factor = [tex]\frac{6.25}{5}[/tex] = 1.25

This represents an enlargement

solve for Z- 2 + 8 = 24 show work​

Answers

Answer:

z = 18

Step-by-step explanation:

[tex]z-2+8=24\\\\z+(-2+8)=24\\\\z+6=24\qquad\text{subtract 6 from both sides}\\\\z+6-6=24-6\\\\z=18[/tex]

Answer:

Z = 18

Step-by-step explanation:

Z- 2 + 8 = 24

Combine like terms

Z +6 = 24

Subtract 6 from each side

Z+6-6 =24-6

Z = 18

Carly's age is 3 more than twice her sister's age, s. Which of the following expressions represents Carly's age?

Answers

Answer:

c = 2s + 3

Step-by-step explanation:

Let c be Carly's age.

Carly is 3 more . . . than twice her sister's age.

We can translate "twice her sister's age" into 2s.

Carly is 3 more than 2s.

We can translate this into:

Carly is 3 + 2s.

c = 2s + 3

Final answer:

Carly's age, given that it's three more than twice her sister's age, can be expressed with the mathematical expression 2s + 3. This translates the stated relationship into an algebraic context.

Explanation:

The question is asking for an expression that represents Carly's age in relation to her sister's age. Given that Carly's age is three more than twice the age of her sister, we can express Carly's age (C) as 2s + 3, where s is her sister's age.

Learn more about Algebraic Expressions here:

https://brainly.com/question/34192827

#SPJ2

Other Questions
is asia east of the us or west of the us? Proofread the following sentence and correct the spelling and grammar errors. There are two errors.Go through reception and turned right at the l entrance. Which term is used to describe a hole played in one stroke under par? Which account has the highest effective annual interest rate? Not necessary but please show how you got your answer.A. Account 1: Interest is compounded quarterly at an annual rate of 4.20%.B. Account 2: Interest is compounded monthly at an annual rate of 4.15%.C. Account 3: Interest is compounded semiannually at an annual rate of 4.10%D. Account 4: Interest is compounded annually at a rate of 4.25%. Is globalization a danger to local cultures? Why, or why not? What are the solution(s) to the quadratic equation x2 25 = 0?O x = 5 and x = -5OX=25 and x = -25O x = 125 and x = -125O no real solution Which of the following is an example of a false causality fallacy?OA. Cora is a good class president, because she is the best presidentwe've ever had.OB. The team started losing after Trish joined, so it must be becauseof herOC. Conroy doesn't like eating spinach, so he must dislike allvegetables.OD. Elliot can either pay for college by working, or he can take out astudent loan and be in debt.SUBMIT What is the surface area of the right cylinder below? 3. Sophie and Jackie each have a collection of baseball cards, Jackie has 5 more cards thanSophie, and together they have 30 cards. By writing and solving an equation, find out howmany cards Sophie owns. Show all of your working out. Complete these equations for the ionization of an Arrhenius acid or base in water. Include the states of the products.HI (aq) ----------------> __________?LiOH (s) -----------------> ________? What event marked the beginning of the great depression Name the three types of population distribution The moment of inertia for a 5500 kg solid disc is 12100 kg-m^2. Find the radius of the disc? (a) 2.111 m (b) 2.579 m (c) 1.679 m (d) 2.574 m (e) 2.098 m (f) 2.457 m During which succession do new species move into an area and colonize it?During____succession, new species move into an area and colonize it. Which new invention helped poorer immigrants from Eastern Europe get to America Writing a quadratic equation given the roots and the leading coefficient6,-4,1 What is the best summary of this story excerpt? What motivates Lillian Jean to tell Cassie all her secrets?her feeling of superiority over Cassieher feeling that Cassie cannot be trustedher need to trick Cassie into liking herher need to make Cassie feel jealous (Easy!)If you could have a pirate ship, what would it look like? Include the size, crew size, color, what rooms you would like, as much detail as you can think of. Dont forget that this would happen hundreds of years ago, so no technology :)15 points ne angle in a triangle has a measure that is three times as large as the smallest angle. The measure of the third angle is 50 degrees more than that of the smallest angle. Find the measure of the LARGEST angle